what is the sum of 8/10 + 60/100

Answers

Answer 1

Answer:

Its 1.4

Step-by-step explanation:

Answer 2

Answer:

[tex]1\frac{2}{5}[/tex]

Step-by-step explanation:

[tex]\frac{8}{10} + \frac{60}{100}[/tex]

first, make sure both equations have the same denominators:

[tex]\frac{8}{10}[/tex] x 10 = [tex]\frac{80}{100}[/tex]

now, take both fractions, add the numerators but keep the denominators:

[tex]\frac{80}{100} + \frac{60}{100}[/tex] = [tex]\frac{140}{100}[/tex]

reduce:

[tex]\frac{140}{100}[/tex] = [tex]1\frac{40}{100} = 1 \frac{4}{10} = 1\frac{2}{5}[/tex]


Related Questions

Why do equivalent ratios form a straight line when graphed?

The vertical distance between points is constant, and the horizontal distance between points increases with each point.

The ratio of the change in y-values to the change in x-values is equivalent for any two points in a graph of equivalent ratios.

The horizontal distance between points is constant, and the vertical distance between points increases with each point.

The y-values of equivalent ratios increase at a constant rate, and the x-values decrease at a constant rate.

Answers

Answer:

D-The y-values of equivalent ratios increase at a constant rate, and the x-values decrease at a constant rate.

Step-by-step explanation:

i just did the quiz


D because the y values increase at a constant rate

Pythagorean Theorem: a^2 + b^2 = c^2

Question 6 options:


No. Since 7^2+9^2≠11^2, the triangle cannot be a right triangle.



Yes. Since 7^2+9^2=11^2, the triangle must be a right triangle.



Yes. Since7^2+9^2≠11^2, the triangle must be a right triangle.



No. Since 7^2+9^2=11^2, the triangle cannot be a right triangle.

Answers

9514 1404 393

Answer:

  No. Since 7^2 + 9^2 ≠ 11^2, the triangle cannot be a right triangle.

Step-by-step explanation:

You may recall that side lengths of 3, 4, 5 make a right triangle. That particular triple has several interesting characteristics. One is that it is the only (reduced) triple that is an arithmetic sequence (has a constant difference between the side lengths). Any right triangle that has sides that differ by a constant amount must be a multiple of 3, 4, 5.

Here, we have sides of length 7, 9, 11—values that differ by 2. They form an arithmetic sequence that is not a multiple of 3, 4, 5, so we know right away they cannot be sides of a right triangle.

__

If we want to use the Pythagorean relation to check, we can see if the equation is true:

  7² + 9² = 11²

  49 + 81 = 121

  130 = 121 . . . . . . Not True

Helpppp mee pleaseee!!

Answers

Answer:




Step by step explanation:

Answer:

# 203 is C

Step-by-step explanation:

5.2×5.2×3.2= 86.528

What is 16x - 13y =7 in slope intercept form

Answers

Answer:

y=12/13x−10/13

Step-by-step explanation:

The answer is y= 16/13x - 7/13

The distance from a driveway to the closest house is 12,672 feet. How far is that in miles? (1 mile = 5280 feet)
A) 2.2 miles
B) 2.4 miles
C) 4.2 miles
D) 12 miles

Answers

B) 2.4 miles
12,672 ft/ 5,280 ft = 2.4miles
B I think because um smart nah I’m playing but i think it’s B

Answer this question ASAP please
I don’t know if you need the second image.

Answers

Member 1 ( Oliver)
Order: chicken tenders, a large green salad with a grilled cheese sandwich with lemonade and for dessert a pecan pie

Member 2 (Susan)
Order: Clam chowder with a BLT and a dot Coca Cola with chocolate pudding

Member 3 (Carl)
Order: Double Cheeseburger with a side of French fries along with the soup of the day and ice tea

Member 4 ( Lana)
Order:
Hummus plate, with a deli sandwich and baked beans and a pecan pie with some lemonade

3x+5y= 10?

A. y =⅗ x+ 10
B. y = 3x− 2
C. y =- ⅗ x +15
D. y = − 5/3 x - 8

Answers

It a or c take a guess
The answer should be y=-3x/5+2

HELP TEN POINTS AND BRAINLEST IF RIGHT TROLL AND I WILL REPORT YOU!!!

Answers

Answer:

C. 1/9 sq yd

Step-by-step explanation:

1/3 * 1/3 = 1/9

Select the statement that makes this comparison correct: 0.45 ________ 0.39.
Question 1 options:

0.39 > 0.45

0.45 > 0.39

0.45 < 0.39

0.39 = 0.45

Answers

Answer:

its c

Step-by-step explanation:

look at the tenths

.39         .45

I think that the correct answer would be the third option. Option C

Part A: What is the approximate y-intercept of the line of best fit and what does it represent? (5 points) Part B: Write the equation for the line of best fit in the slope-intercept form and use it to predict the number of matches that could be won after 13 months of practice. Show your work and include the points used to calculate the slope

Answers

Answer:

Well I can't do it if you don not show the graph :(

Step-by-step explanation:

Answer:

Let's actually find the line of best fit...

m=(nΣyx-ΣyΣx)/(nΣx^2-ΣxΣx)

m=(11*836-130*55)/(11*385-3025)

m=2046/1210  

m=93/55

b=(Σy-93Σx/55)/n

b=(55Σy-93Σx)/(55n)

b=(7150-5115)/(55*11)

b=185/55, so the line of best fit is:

y=(93x+185)/55

A)  The approximate y-intercept (the value of y when x=0) is 185/55≈3.36.

Which means that those who do not practice at all will win about 3.36 times

B) y(13)=(93x+185)/55

y(13)≈25.34

So after 13 months of practice one would expect to win about 25.34 times.

what is y
help please

Answers

y= 13.
EXPLANATION.
According to the table,
X is 5.
And y is in the column on the right,
you add 2 for each number.
11 + 2 = 13.
HAVE A GUD DAY

explain how to answer the word problem below.

3 feet = 1 yard
John has 476 feet of yarn and he needs to know how many yards he has. How many yards of yarn does John have. (solve and explain)

Answers

You get 158.667 yards of yarn

Explanation-You dived the length value by 3 since 3 feet = 1 yard
158.7 this is because you have to divide 476 by 3 because each 3 feet of yarn is a yard. Hope that makes sense I’m not very good at explaining.

The radius of a circle is 3 in. Find its area in terms of π.


Please help :(

Answers

Answer:

A≈28.27

Step-by-step explanation:

Answer:

3

Step-by-step explanation:

While at the beach, daniel buys lunch for his family from a food stand. he buys one hot dog for $2.50 and 3 hamburgers. if he spent $13 total, write and solve the equation to find H, the amount each hamburger cost.

Answers

Answer:

3.5$

Step-by-step explanation:

simple.

Answer:13 = 2.50+3h

Step-by-step explanation:

H = $3.50

What operation should you perform first when you evaluate this expression? (15 x 5 x 4) x 3 *
Answers:
15x5
5x4
4x3

Answers

15*15. You’d still get the same answer if you do 5*4 and then 15

Do any of you have a made-up pokemon? Give Me Name, Hight, Type, Category, And All of That Stuff.

Answers

I've got a Pokémon is called bashy smashy it looks like a crocodile with a massive spicy steel ball at the end it's signature move is bashy ball it's a rock and steel type but it can learn dark type moves it's 4-ft 11 and weighs 80 lb

i need help pls
and explain aslo

Answers

Answer:

7

Step-by-step explanation:

Answer=7.536m3
Volume divided by 6.4m

Help, please!
The angle measurements in the diagram are represented by the following expressions.





Solve for x and then find the measure of ∠B

Answers

B=A the are the same angle B and A

The model car is 3 inches long. On the box, it says the scale of the model is 1/42. What is the length of the actual car in feet? (convert your answer from inches to feet.)

Answers

Answer:

10 feet 6 inch

Step-by-step explanation:

3 times 42 is 126. 126 to feet is 10 ft 6 inch

10 ft and 6in hope this helps

Find the value of the x

Answers

Answer:

The answer should be 77, but the triangle in the question seems off. Read the explanation.

Step-by-step explanation:

To find the exterior angle in this problem, you can use the exterior angle theorem.
Interior angle + interior angle = exterior angle
25 + 52 = 77
You can double-check this answer by figuring out the third interior angle.
Since the sum of all the interior angles of a triangle should be 180, you can determine that
25 + 52 + y =180
Y= 103
or, since a straight line is always 180 degrees (Y+X=180)
25+52+(180-x)=180

X= 77
The reason why I think the question looks weird is since 77 is an acute angle, and even though it's the only answer that makes sense in this question, angle X is clearly an obtuse angle, as shown in the picture.



What is the approximate area of the two congruent red right triangles?


A. 17 in²

B. 31 in²

C. 33 in²

D. 51 in²

Answers

Answer:

A. 17in

Step-by-step explanation:

Answer: B. 31 in²

Step-by-step explanation:

Look who I recreated

Please answer 12. How ever gets it right will get brainlyest

Answers

Answer:

-6.5

Step-by-step explanation:

-18-2-6+14=26

-26/4=-6.5

Answer: -3 yards

Step-by-step explanation:

-2+14+-18+-6 = -12/4 = -3

Does this check out?
Its for pre-algebra.

Answers

Answer:yes

Step-by-step explanation:

Answer:

yes, that is correct, good job!

Mrs.Smith has 3 boys to every 5 girls in her class. What percent of her class is boys?
Please help me! Serious answers only please!

Answers

Answer:

1/4

25 percent

Step-by-step explanation:

Answer:

60% of the class is boys.

Step-by-step explanation:

Me not being a complete id.iot-

Answers

Answer:

im pretty sure the answer is d

Step-by-step explanation:

A line that includes the points (4,j) and ( – 8, – 9) has a slope of 16 . What is the value of j?

Answers

The answer is j= -8, if the slope is 16/1 then -8 goes up 16 to 4 and -9 goes over 1 to -8

Clare estimates that her brother is 4 feet tall. When they get measured at the doctor’s office, her brother’s height is 4 feet, 2 inches.

1. Should Clare's or the doctor's measurement be considered the actual height? Explain your reasoning.

2. What was the error, expressed in inches?

3. What was the error, expressed as a percentage of the actual height?

Answers

Answer:

DAMMMMMMMMMMM HE SHORT

Step-by-step explanation:

please help me the best that you can. this is my last grade for this class and I really need to pass.

Answers

Answer:

v = 52 m/s

Step-by-step explanation:

V= 52 m/s I think I believe so

PLEASE HELP- THIS IS DUE AT 2PM PLSSSS HELPPPP QUICKKKK!! WILL MARK BRAINLIEST


There are three parts to this problem. Solve each percent word problem and show your setup for each one.



You have r rare coins, consisting of p pennies and n nickels.

1. p is 20% of 190. How many pennies do you have?


2. 190 is 200% of r. How many rare coins do you have?


3. n is 60% of r. How many nickels do you have?

Answers

Answer:

1. p=38 pennies. 2 r= 95 rare coins.  3. n=57 nickels

Step-by-step explanation:

1. how you get it is .20 x 190

2. 190/2 =2/3 x r

3. .60 x 95

Which best describes the mean of a set of data?

A. The difference between the lowest and the highest values in a set of data.
B. The average of a set of data.
C. The value that occurs most frequently in a set of data
D. The middle value in a set of data.

Answers

The answer is B it is correct because i hot a 100
B. the average of a set of data.
Other Questions
the indicated gene codes for a protein made up of the amino acid What do proteins and amino acids do for your brain? In your own words, explain the motivation of slaves in the civil war. J.T. wants to use a more appropriate transition in sentence 11. Which of the following can best replaceFurthermore in this sentence?a. for example b. in conclusion c. overalld. instead Which statements correctly identify chart of Earth and Jupiter? can anyone help me with this plz help me pls and show work 8th grade math it takes 4.5 ounces to my a copper pipe fitting. American pipe estimates they produce an average of 25.4 fittings per week. How much copper will it take to make fittings for the whole year? Posted the next question! A person standing on level ground is 3020 feet away from the foot of a building. If the height of the building is 2534 feet, find the angle of elevation formed at the point where the person is standing to the nearest degree HELPP PLEASEEEE FASTTTTBoth parents in the example carry the dominant and therecessive alleles for a trait (heterozygous). What is theprobability of the offspring expressing the recessivephenotype? Does a rotation preserve congruence What was the name of the Judicial Branch under the Articles ofConfederation? Find the difference using fraction bars or a paper and pencil. Write your answer in simplest form. 7/10 - 1/2 help asap!! im on a test 7. Immediately after leaving the right ventricle, blood enters whichstructure of the circulatory system? * PLEASE HELP ASAPThree gases are at the same temperature and so have the same kinetic energy . The atomic mass of gas 1 is 10amu the atomic mass of gas 2 is 25 and atomic mass of gas 3 is 8. Which gas will have the greatest velocity ? ok pls do this please and thank you HELP PLS!!! (PSYCHOLOGY) I'LL GIVE YOU BRAINLIEST IF UR RIGHT!!Nate is a teacher who has been in the dassroom for 25 years. When a new teacher is hired, he offers to be a mentor because he loves helping new teachers get inspired about working with children. He wants to help make sure that the teaching profession remains high quality. How would Erikson MOST likely describe Nate's efforts? A.) Nate is working en generativity. B.) Nate is working on ego integrity.C.) Nate is working on intimacy. D). Nate is working on acceptance Katherine, a 30-year-old woman, has had difficulty recalling certain time frames from her childhood, and she becomes anxious when returning to her grandparents home to visit. Katherine is most likely suffering from __________.A.dissociative amnesiaB.demoralizationC.dissociative identity disorderD.dissociative fugue what does 9a^2 + 2b^2 if a=4 and b=7?